PTA.S1.Q23 - Health association and critic re certain studies

McBeck418McBeck418 Member
edited January 2021 in Logical Reasoning 500 karma

I am really confused by this question, partially because I'm not sure what the question stem is asking us to do, but then also, I don't get why the answer is E. I think the q-stem wants us to consider one of the answer choices as the conclusion put out by the health association. I just feel very lost by this question.

The critic seems to undermine B, but I guess its wrong because its not something the Health Association would say? A-D don't seem to be things that the critic would undermine. They seem irrelevant.

The only way I look at E is that since volunteers are able to suceed, something stronger than a volunteer (being told by a physician) would also have to succeed and the critic's response is still applicable?

Comments

  • FautApprendreLSAT1FautApprendreLSAT1 Core Member
    edited January 2021 314 karma

    Hey,
    Although I got it right, I too, was a bit confused by the wording of this question at first. It took me two whole minutes to get it. Basically, it is asking you to identify an answer that would be in agreement with the Health Association's argument (that based on studies, people who eat meat can change their diets to meatless ones and not suffer as a result) and in disagreement with the Critic's argument (that the disposition of the people in the studies made them an inaccurate representation of people as a whole).

    Anyway, I chose E because it was the only which I could see that the two sides have clear opposite opinions on. The Critic, based on his response, would disagree with answer E--perhaps stating that, we cannot make such a generalized extrapolation based on an argument premised on faulty studies. Meanwhile, the Health Association would agree with answer E since it is in line with their initial conclusion that "people who eat meat can change their diets to meatless ones and not suffer as a result..." I hope this makes sense.

  • AlexJ_LSATAlexJ_LSAT Core Member
    141 karma

    Thank you, this was an excellent explanation.

  • FautApprendreLSAT1FautApprendreLSAT1 Core Member
    314 karma

    @AlexJ_LSAT Great, happy to help!

  • McBeck418McBeck418 Member
    edited January 2021 500 karma

    @Isaaa---189 Thank you for your response. I feel like i was half way there, but not seeing the connection. Your explanation makes it at lot clearer now.

  • axbSunDevaxbSunDev Member
    edited January 2021 256 karma

    This is a pretty hard question but hopefully I can help break it down. The stem is a bit complicated but it is saying that one of the answer choices is using the HA's argument as support, and we need to find the conclusion that can best be weakened by the critic's argument. So which AC is the best target that the critics can undermine. AC E is basically saying most (more than 51%) of patients, if told to exclude meat, would be just fine and would succeed, meaning they would not have "ill effects" like the HA argument states. But the critic's argument undermines this statement because he states that 1) the study was flawed 2) many of these subjects were not fully successful. So how could a doctor actually tell their patient that information that they would be successful if the study they are using to back it up was flawed and the subjects were not even truly successful? The critic's argument makes AC E a very flawed diagnosis.

    Hopefully this helps in some way! This question is a bit hard to articulate.

Sign In or Register to comment.